Вы находитесь на странице: 1из 37

Chapter 1 The properties of gases

Exercises
1.2(b) (a)Could 25 g of argon gas in a vessel of volume 1.5L exert a pressure of 2.0 bar at 30C if it
behaved as a perfect gas? If not, what pressure would it exert?(b) what pressure would it exert if it behaved
as a van der Waals gas?
Solution: (a) The perfect gas law is
pV = nRT
implying that the pressure would be
nRT
p=
V
All quantities on the right are given to us except n, which can be computed from the given mass
of Ar.
25 g
n= = 0.62 6 mol
39 .95 gmol 1
( 0.626 mol ) ( 8.31 10 2 Lbark -1mol -1 ) (30 + 273k)
so p = = 10 . 5bar
1.5L
not 2.0 bar.
(b) The van der Waals equation is

RT a
p= 2
Vm b Vm

( 8.31 10 2 LbarK 1 ) ( 30 + 273 )K


so p = = 10 .4bar
(1.5L / 0.62 6mol ) 2

1
1.5(b) A sample of hydrogen gas was found to have a pressure of 125 kPa when the temperature was 23C,
what can its pressure be expected to be when the temperature is 11C?
Solution:The relation between pressure and temperature at constant volume can be derived from the perfect
gas law

pi p
pV = nRT so p T and = f
Ti Tf

The final pressure, then, ought to be

p iT f (125 kPa ) (11 + 273 )K


pf = = = 120 kPa
Ti (23 + 273)K

2
1.7(b) The following data have been obtained for oxygen gas at 273.15K. Calculate the best value of the
gas constant R from them and the best value of the molar mass of O2.
P/atm 0.750 0.500 0.250 000
000 000
Vm/Lmol-1 29.8649 44.8090 89.6384
/(gL-1) 1.07144 0.714110 0.356975
Solution:

All gases are perfect in the limit of zero pressure. Therefore the extrapolated value of pV m / T will give

the best value of R.


m
The molar mass is obtained from pV = nRT RT
M
m RT RT RT
Which upon rearrangement gives M = =
V p p p

The best value of M is obtained from an extrapolation of
p versus p to p= 0the intercept is M/RT
Draw up the following table
p / atm ( pV m / T ) /( L atm K -1 mol -1 ) ( / p ) /( g L-1 atm -1 )
0.750 000 0.082 0014 1.428 59
0.500 000 0.082 0227 1.428 22
0.250 000 0.082 0414 1.427 90

pVm
From Fig. 1.1(a) = 0.082 0615 L atm K -1 mol -1
T p =0
-

From Fig. 1.1(b)


p = 1.42755 g L 1 atm -1

p =0

-
M = RT
p
= ( 0.082 0615 L atm K -1 mol -1
) ( 273 .15 K) (1.42755 g L 1 atm -1
)
p =0
1
= 31 .9987 g mol

The value obtained for R deviates from the accepted value by 0.005 per cent. The error results from the fact
that only three data points are available and that a linear extrapolation was employed. The molar mass,
however, agrees exactly with the accepted value, probably because of compensating plotting errors.
THE PROPERTIES OF GASES

3
1.8(b) At 100C and 120 Torr, the mass density of phosphorus vapour is 0.6388 Kgm-3 . What is the
molecular formula of phosphorus under these conditions?
Solution:The mass density is related to the molar volume Vm by
M
Vm

Where M is the molar mass. Putting this relation into the perfect gas law yields
pM
pV m = RT so = RT

Rearranging this result gives an expression for M; once we know the molar mass, we can divide by
the molar mass of phosphorus atoms to determine the number of atoms per gas molecule
RT 62 .364 LTorr K -1 mol -1 [( 100 + 273 )K] (0.6388gL -1
) 1
M = = = 124 gmol
p 120 Torr
The number of atoms per molecule is

124 gmol 1
= 4.00
31 .0gmol 1

4
1.10(b) A gas mixture consists of 320 mg of methane, 175 mg of argon, and 225 mg of neon. The partial
pressure of neon at 300K is 66.5 Torr. Calculate(a) the volume and (b) the total pressure of the mixture.
Solution: (a) The volume occupied by each gas is the same, since each completely fills the container. Thus
solving for V from eqn 14 we have (assuming a perfect gas)
nJ RT 0.225 g
V = n Ne = -1
pJ 20.18gmol
= 1.11 5 10 2 mol, pNe = 66 .5Torr, T = 300 K
(1.115 10 2 mol) ( 62 .36 LTorr K -1 mol -1 ) ( 300 K)
V = 3.13 7 L = 3.14 L
66.5Torr
(b) The total pressure is determined from the total amount of gas, n=nCH4+nAr+nNe.
0.320 g 0.175 g
nCH4 = 1
= 1.995 10 - 2 mol nAr = 1
= 4.38 10 3 mol
16.04 g mol 39.95 g mol
n = (1.995 + 0.438 + 1.115) 10 2 mol = 3.548 10 2 mol
nJ RT ( 3.548 10 2 mol) ( 62.36LTorr K -1 mol -1 ) ( 300K)
p= [1] = = 212 Torr
V 3.137L

5
1.13(b) Determine the ratios of (a) the mean speeds, (b)the mean kinetic energies of He atoms and Hg
atoms at 25C.
Solution:(a) The mean speed of a gas molecule is
1
8 RT 2
c =
M
1 1
c ( He ) M ( Hg ) 2
200 .59 2
so = = = 7.079
c ( Hg )
M ( He ) 4.003

1
(b) The mean kinetic energy of a gas molecule is mc 2 ,where c is the root mean square speed
2
1
3 RT 2
c =
M

1
So mc 2 is independent of mass, and the ratio of mean kinetic energies of He and Hg is 1.
2

6
1.14(b) The best laboratory vacuum pump can generate a vacuum of about 1 nTorr. At 25C and assuming
that air consists of N2 molecules with a collision diameter of 395 pm, calculate (a) the mean speed of the
molecules, (b) the mean free path, (c) the collision frequency in the gas.
Solution: (a) The mean speed can be calculated from the formula derived in Example 1.6
1 1
8 RT 2 8 8.314 JK -1 mol -1 ) ( 298 K) 2
c = = c =

= 4.75 10 2 ms 1
M ( 28 .02 10 3 kg mol -1

kT
(b) The mean free path is calculated from = 1
[ 33]
2 p 2

With = d 2 = ( 3.95 10 10 m) 2 = 4.90 10 19 m 2


Then,

1.381 10 23 JK -1 ) ( 298 K)
= = 4 10 4 m
1 1 atm 1.013 10 5
Pa
2 2
( 4.90 10 19 m 2 ) (1 10 9 Torr)
760 Torr 1 atm
(c) The collision frequency could be calculated from eqn 31, but is most easily obtained from eqn 32, since
c 4.75 c
and c have already been calculated z = = = 1 10 2 s 1
4.46 10 4 m
Thus there are 100s between collisions, which is a very long time compared to the usual timescale
apparatus used to generate the very low pressure.

7
1.16(b) At an altitude of 15 km the temperature is 217 K and the pressure 12.1 kPa. What is the mean free
path of N2 molecules? (=0.43nm2)
Solution: The mean free path is

kT 1.381 10 23 JK 1 ) (217K)
= = 4.1 10 7 m
1
2 2 p 2
1
2
[0.43 (109
]
m)2 (12.1 103 Pa atm1 )
1.17(b) How many collisions per second does an N2 molecule make at an altitude of 15 km?(See Exercise
1.16b for data.)
1

Solution: Obtain data from Exercise 1.17(a) is z = 16 p


2

mkT

Substituting = 0.43 nm 2 , p = 12 .1 10 3 Pa , m = ( 28 .02 u ), andT = 217 K


we obtain
18
4 (0.43 10 m ) (12.1 10 Pa)
2 3
z=
[ (28.02) (1.6605 10 27
( )
kg) 1.381 10 23 J K 1 ( 217 K ) ] 1
2

=9.9108s-1

8
1.21(b) Estimate the critical constants of a gas with van der Waals parameters a=1.32 atmL2mol-2 and
b=0.0436Lmol-1.
Solution: The critical constants of a van der Waals gas are

Vc = 3b = 3( 0.0436 Lmol 1 ) = 0.131 Lmol 1


a 1.32 atm L2 mol -2
pc = = = 25 .7 atm
27 b 2 27 ( 0.08206L atm K 1 )2
8a 8(1.32atmL 2 mol 2 )
and Tc = = = 109 K
27 Rb 27(0.08206 L atm K 1 ) (0.0436 L mol 1 )

1.22(b) A gas at 350K and 12 atm has a molar volume 12 per cent larger than calculated from the perfect
gas law. Calculate (a) the compression factor under these conditions and (b) the molar volume of the gas.
Which are dominating in the sample, the attractive or the repulsive forces?
Solution: The compression factor is

pV m Vm
Z = =
RT Vm . perfect
(a) Beacuse Vm =Vm . perfect +0.12 Vm . perfect =0.12 Vm . perfect we have
Z =1.12 Repulsive forces dominate.
(b) The molar volume is
RT
V = (1.12) Vm . perfect = (1.12)
p


(0.08206 L atm K -1 mol -1
) (350 K)
V = (1.12)

= 2.7 L mol
1

12 atm

9
1.24(b) The density of water vapour at 1.00 bar and 383 K is 0.5678 kg m -3.(a) Determine the molar volume
Vm of water and the compression factor Z, from these data. (b) Calculate Z from the van der Waals equation
with a=5.536L2atm mol-2 and b=0.03049L mol-1.
Solution: (a)
M 18 .015 g mol 1
Vm = = = 31 .72 8 L mol 1
0.5678 g mol 1
pVm (1.00 bar) (31.72 8 L mol 1 )
Z = = = 0.9963
RT (0.083145 L bar K 1 mol 1 ) (383 K)
RT a
(b) Using p = 2 and subsititut ing into the expression for Z above we get
Vm b Vm
Vm a
Z= -
Vm b Vm RT
31.72 8 L mol -1 5.536L 2 atm mol 2
= = 0.995 4
31 .728 L mol -1 0.03049 L mol -1 31.72 8 L mol -1 0.03049L mol -1
Comment. Both values of Z are very close to the perfect gas value of 1.000, indicating that water vapour is
essentially perfect at 1.00 bar pressure.

10
1.25(b) At 300 K and 20 atm, the compression factor of a gas is 0.86. Calculate (a) the volume occupied by
3.2 mmol of the gas under these conditions and (b) an approximate value of the second virrial coefficient B
at 300 K.
Solution:
pV m
The molar volume is obtained by solving Z = [ 34 ] , for Vmwhich yields
RT
ZRT 0.08206 L atm K 1 ) (300 K)
Vm = = = 1.059 L mol 1
p 20 atm
(a) Then, V = nV m = (8.2 10 - 3 mol) (1.0 59 L mol -1 ) = 8.7 10 - 3 L = 8.7 mL
(b) An approximat e value of B can be obtained from eqn 36 by truncation of the series expansion
pV m
B = Vm 1 = Vm ( Z-1)
RT
= (1.0 59 L mol -1 ) ( 0.86 1) = 0.15 L mol -1

11
1.27(b) The critical constants of ethane are pc=45.6 atm, Vc=148 cm3 mol-1, and Tc=305.4 K. Calculate the
van der Waals parameters of the gas and estimate the radius of the molecules.
Solution:
The critical volume of a van der Waals gas is
Vc = 3b

so b =
1
3
1
( )
Vc = 148 cm 3 mol 1 = 49 .3cm 3 mol 1 = 0.0493 L mol 1
3
By interpreti ng b as the cxcluded volume of a mole of spherical molecules, we can obtain an estimate
of molecular size. The centres of spherical particles are excluded from a sphere whose radius is the
diameter of those spherical particles (i.e., twice their radius); that volume times the Avogadro constant
is the molar excluded volume b
1
4 ( 2r ) 3 3
b = N A so r = 1 3b
3
2 4 N A

1
1 3(49.3 cm 3 mol 1 ) 3
r = = 1.347 10 8 cm = 1.94 10 -10 m
2 4(6.02 10 23 mol 1 )
The critical pressure is
a
pc =
27 b 2
(
so a = 27 pc b 2 = 27 ( 48 .20 atm ) 0.0493 L mol 1 ) 2
= 3.16 L2 atm mol 2
But this problem is overdeterm ined. We have another piece of informatio n
8a
Tc =
27 Rb
According to the constants we have already determined , Tc should be

Tc =
(
8 3.16 L2 atm mol 2 )
= 231 K
( )
27 0.08206 L atm K 1 mol 1 (0.0493 L mol 1 )
However, the reported Tc is 305.4 K, suggesting our computed a/b is about 25 per cent lower than it
should be.

12
1.29(b) Suggest the pressure and temperature at which 1.0 mol of (a) H 2S, (b)CO2, (c) Ar will be in states
that correspond to 1.0 mol N2 at 1.0 atm and 25C.
Solution:

States that have the same reduced pressure, temperatur e, and volume are said to correspond . The
reduced pressure and temperatur e for N2 at 1.0 atm and 25 C are
p 1.0 atm T (25 + 273) K
pr = = = 0.030 and Tr = = = 2.36
pc 33.54 atm Tc 126.3 K
The correspond ing states are
(a) For H 2S
p = pr pc = (0.030) (88.3 atm) = 2.6 atm
T = TrTc = (2.36) (373.2 K) = 881 K
(Critical constants of H 2 S obtained from handbook o f chemistr y and phys ics. )
(b)For CO 2
p = pr pc = (0.030) (72.85 atm) = 2.2 atm
T = TrTc = (2.36) (304.2K) = 718 K
(c) For Ar
p = pr pc = (0.030) (48.00atm) = 1.4 atm
T = TrTc = (2.36) (150.72K) = 356 K

13
1.30(b) A certain gas obeys the van der Waals equation with a=0.76 m6 Pa mol-2. Its volume is found to be
4.0010-4 m3 mol-1 at 288 K and 4.0Mpa. From this information calculate the van der Waals constant b.
What is the compression factor for this gas at the prevailing temperature and pressure?
Solution:
The van der Waals equation is
RT a
p= 2
Vm b Vm
which can be solved for b
RT
b = Vm = 4.00 10 4 m 3 mol 1
a
p+ 2
Vm
The compressio n factor is

Z=
pV m
=
( ) (
4.0 10 6 Pa 4.00 10 4 m 3 mol 1 )
= 0.67
RT ( )
8.3145 J K 1 mol 1 ( 288 K )

14
Problems
Numerical problems
1.4 A meterological balloon had a radius of 1.0 m when released at sea level at 20C and expanded to a
radius of 3.0 m when it had risen to its maximum altitude where the temperature was -20C. What is the
pressure inside the balloon at that altitude?
Solution:
p fVf piVi
pV = nRT[12] implies that, with n constant, =
Tf Ti
Vi T f
Solving for p f , the pressure at its maximum altitude, yields p f = pi
V f Ti
4 3 4 3
Substituting Vi = ri and V f = rf
3 3
( )
4 ri 3
pf = 3 3
T 3
f p = ri T f p
( )
4 r
3 f
T
i
i r
f Ti
i

3
1.0m 253 K
( 1.0 atm) = 3.2 10 atm
2
=
3.0m 293 K

15
1.10 A vessel of volume 22.4 L contains 2.0 mol H2, and 1.0 mol N2 at 273.15 K initially. All the H2 reacted
with sufficient N2 to form NH3. Calculate the partial pressure and the total pressure of the final mixture.4
Solution:
We assume that no H2 remains after the reaction has gone to completion . The balanced equation is
N 2 + 3H 2 2NH 3

We can draw up the following table

N2 H2 NH3 Total
Initial amount n n' 0 n + n'
Final amount 1 0 2 1
n n' n' n + n'
3 3 3
Specifically 0.33mol 0 1.33 mol 1.66 mol
Mole fraction 0.20 0 0.80 1.00

p=
nRT
= ( 1.66mol )
( )
8.206 10 2 L atm K 1 mol 1 ( 273.15 K )
= 1.66 atm
V 22.4 L
p(H 2 ) = x (H 2 ) p = 0
p(N 2 ) = x (N 2 ) p = ( 0.20 ( 1.66 atm ) ) = 0.33 atm
p(NH 3 ) = x (NH 3 ) p = ( 0.80 ) ( 1.66 atm ) = 0.33 atm

16
1.15 Calculate the molar volume of chlorine gas at 350 K and 2.30 atm using (a) the perfect gas law and (b)
the van der Waals equation. Use the answer to (a) to calculate a first approximation to the correction term
for attraction and then use successive approximations to obtain a numerical answer for part(b).
Solution:
RT (8.206 10 2 L atm K 1 mol 1 )(350 K)
(a)Vm = = = 12 .5 L mol 1
p 2.30 atm
RT a
(b)From p= [ 39b ], we obtain Vm = RT + b[ rearrange3 9b ]
Vm b Vm 2
p + a2
Vm

Then , with a and b from Table 1.6
(8.206 10 2 L atm K 1 mol 1 )(350 K)
Vm (
+ 5.622 10 2 L mol 1 )
6.579L atm mol
2 2
(2.30 atm) + 1 2

(12.5 L mol )
28 .7 2L mol 1

2.34
( )
+ 5.622 10 2 L mol 1 12 .3 L mol 1

Substituti on of 12.3 L mol 1 into the denominato r of the first expression again results in
Vm = 12 .3 L mol 1 , so the cycle of approximat ion may be terminated .

17
Chapter 2 The First Lawthe concepts
Exercises
2.4 (b) A sample consisting of 2.00 mol He is expanded isothermally at 22 from 22.8 L to 31.7 L (a)
reversibly, (b) against a constant external pressure equal to the final pressure of the gas, and (c)
freely(against zero external pressure). For the three processes calculate q,w,U,and H.
SolutionFor a perfect gas at constant temperature
U = 0 so q = -w
For a perfect gas at constant temperature, H is also zero
dH = d( U + pV )
we have already noted that U does not change at constant temperature; nor does pV if the gas obeys
Boyles law. These apply to all three cases below.
(a) Isothermal reversible expansion

Vf
w = nRT ln
Vi
31.7 L
= -(2.00 mol) (8.3145J K -1 mol -1 ) (22 + 273) K ln = 1.62 10 3
22.8 L
q = w = 1.62 10 3 J

(b) Expansion against a constant external pressure

w = pex V

Where pex in this case can be computed from the perfect gas law

pV = nRT
(2.00 mol) (8.3145J K -1 mol -1 ) (22 + 273) K
so p = (1000 Lm 3 ) = 1.55 10 5 Pa
31.7 L
(c)
- (1.55 10 5 Pa) (31.7 - 22.8)L
and w = = 1.38 10 J3

1000 L m 3
q = w = 1.38 10 3 J
Free expansion is expansion against no force, so w = 0 , and q = -w = 0 as well.

18
2.6(b) A sample of argon of mass 6.56 g occupies 18.5 L at 305 K. (a) Calculate the work done when the
gas expands isothermally against a constant external pressure of 7.7 kPa until its volume has increased by
2.5 L. (b) Calculate the work that would be done if the same expansion occurred reversibly.
Solution :

( a) w = pex V =
( )
7.7 10 3 Pa ( 2.5L )
= 19 J
1000 L m 3
Vf
(b) w = -nRT ln
Vi

= -
6.56 g
( )
8.3145 J K -1 mol -1 ( 305 K ) ln
( 2.5 + 18.5 ) L
-1
39.95 g mol 18.5 L
= -52.8 J

19
2.8(b) A sample of 2.00 mol CH3OH(g) is condensed isothermally and reversibly to liquid at 64. The
standard enthalpy of vaporization of methanol at 64 is 35.3 kJ mol-1. Find q,w,U,and H for this
process.

(
Solution: q = H = n( vap H ) = ( 2.00 mol ) 35 .3K J mol
1
= 70 .6 kJ)
Because the condensation also occurs at constant pressure, the work is

w = pex dV = pV

The change in volume from a gas to a condensed phase is approximately equal in magnitude to the volume
of the gas

(
w p- Vvapour = nRT = ( 2.00mol ) 8.3145 kJ K -1 mol -1
) ( 64 + 273 ) K = 5.60 10 3
J
U = q + w = (- 70 .6 + 5 .60 )kJ = -65 .0kJ

20
2.11(b) The constant-pressure heat capacity of a sample of a perfect gas was found to vary with temperature
according to the expression Cp,m/(JK-1)=20.17+0.4001(T/K). Calculate q,w,U,and H for 1.00 mol when
the temperature of 1.00 mol of gas is raised from 0 to 100 (a) at constant pressure (b) at constant
volume.
Solution
(a)At constant pressure
[ 20 .17 + ( 0.4001 )T / K ]dT J K 1
100 +273K
q = CpdT =
0 +273K

1
(
)
= ( 20 .17 ) T + ( 0.4001 ) T 2 / K J K 1 273K
2
373K


1
(
)
= ( 20 .17 ) ( 373 273 ) + ( 0.4001 ) 373 2 273 2 J = 14 .9 10 3 J = H
2

( )
w = p = nRR = (1.00 mol ) 8.3145J 1 mol 1 (100 K ) = 831 J
U = q + w = (14.9 - 0.831 ) kJ = 14.1 kJ
(b) U and H depend only on temperatur e in perfect gases. Thus, H = 14 .9kJ and U = 14 .1kJ
as above. At constant volume, w = 0 and U = q , so q = +14 .1kJ

21
2.13(b) A sample of nitrogen of mass 3.12 g at 23.0 is allowed to expand reversibl y and adiabatically
from 400 mL to 2.00 L. What is the work done by the gas?
Solution: Reversible adiabatic work is

w = CV T = n(C p ,m R ) (T f Ti )

Where the temperatures are related by [solution to Exercise2.12b]


1c
V

Tf = Ti i

V

f

C C R (29.125 8.3145) J K 1 mol 1
where c = v,m = p,m = = 2.503
R R 8.3145 J K 1 mol 1
1 2.503
400 10 3 L
So Tf = [ ( 23.0 + 273.15 ) K ] = 156 K
2.00 L
3.12 g
and w = ( 29.125 8.3145 ) J K 1 mol 1 ( 156 296 ) K = 325 J
-1
28.0 g mol

22
2.15(b) Calculate the final pressure of a sample of water vapour of mass 1.4 g that expands reversibly and
adiabatically from an initial temperature of 300 K and volume 1.0 L to a final volume of 3.0 L. Take =1.3.

Solution: For reversible adiabatic expansion


V
so p f = pi i

p f v f = piVi
V
f

We need pi , which we can obtain from the perfect gas law

nRT
pV = nRT so p=
V
1.4 g

18 g mol 1
( )
0.08206 L atm K 1 mol 1 ( 300 K )
pi = = 1.9 atm
1.0 L
1.3
1.0 L
p f = (1.9 atm ) = 0.46 atm
3.0 L

23
2.19(b) When 2.0 mol CO2 is heated at a constant pressure of 1.25 atm, its temperature increases from 250
K to 277 K. Given that the molar heat capacity of CO2 at constant pressure is 37.11 JK-1 mol-1, calculate
q,U,and H .
Solution:

(
H = q p = C p T = nC p,m T = ( 2.0 mol ) 37.11 J K 1 mol 1
) ( 277 250 ) K
1
= 2.0 10 J mol
3

H = U + ( pV ) = U + nR R so U = H nR R
U = 2.0 10 J mol
3 1
(
( 2.0 mol ) 8.3145 J K 1 mol 1
) ( 277 250 ) K = 1.6 10 3
J mol 1

24
2.21 (b) A sample consisting of 2.5 mol of perfect gas at 220 K and 200 kPa is compressed reversibly and
adiabatically until the temperature reaches 255 K. Given that its molar constant-volume heat capacity is
27.6 JK-1 mol-1, calculate q,w,U,and H,and the final pressure and volume.
Solution: For adiabatic compression, q=0 and

( )
w = CV T = ( 2.5 mol ) 27.6 K J 1 mol 1 ( 255 220 ) K = 2.4 10 3 J
U = q + w = 2.4 10 J 3

H = U + ( pV ) = U + nRR
( )
= 2.4 10 3 J + ( 2.5 mol ) 8.3145 J K 1 mol -1 ( 255 220 ) K = 3.1 10 3 J

The initial and final states are related by


c
T
V f T f = V i Ti so V f = V i i
c c
T
f
C 27.6 J K -1 mol 1
where c = V,m = = 3.32
R 8.314 J K 1 mol 1
nRT i 2.5 mol 8.3145 J K 1 mol 1 220 K
Vi = = = 0.022 9 m 3
pi 200 10 3 Pa

( )
3.32
220 K
V f = 0.022 9 m 3
= 0.014 m 3 = 14 L
255 K
nRT f 2.5 mol 8.3145 J K 1 mol 1 255 K
pf = = 3
= 3.8 10 5 Pa
Vf 0.014 m

25
2.24(b) Consider a system consisting of 3.0 mol O2 (assumed to be a perfect gas) at 25 confined to a
cylinder of cross-section 22 cm2 at 820 kPa. The gas is allowed to expand adiabatically and irreversibly
against a constant pressure of 1.0 atm. Calculate q,w,U,and H and T when the piston has moved 15
cm.
Solution: In an adiabatic process, q=0. Work against a constant external pressure is

cm ) ( 22 cm )
( ) (15(100
2
w = pex V = 110 10 3 Pa 1 3
= 36 J
cm m )
U = q + w = 36 J
w = CV T = n(C p,m -R ) T so
w
T =
n( C p,m -R )
36 J
= = 0.57 K
( 3.0 mol ) ( 29 .355 8.3145 ) J K -1 mol -1
H = U + ( pV ) = U + nRR
= 36 J + ( 3.0 mol ) (8.3145 J K -1 mol -1 ) ( 0.57 K ) = 50 J

26
2.27(b) The standard enthalpy of formation of phenol -165.0 kJ mol-1. Calculate its standard enthalpy of
combustion.
Solution: The reaction is
C 6 H 5 OH + 7O 2 6CO 2 + 3H 2 O
c H = 6 f H ( CO2 ) + 3 f H ( H 2 O ) f H ( C 6 H 5 OH ) 7 f H ( O 2 )
= [ 6( 393 .51 ) + 3( 285 .83 ) ( 165 .0 ) 7 ( 0 ) ]K J mol 1
= 3053.6 kJ mol 1

27

2.29(b) From the following data, determine f H for diborane, B2H6(g), at 298 K:

(1) B 2 H 6 ( g ) + 3O 2 ( g ) B 2 O 3 ( s) + 3H 2O( g ) r H = 1941 kJ mol 1

( 2) 2B ( s) + 3 O 2 ( g ) B 2O 3 ( s) r H = 2368 kJ mol 1
2
( 3 ) H 2 ( g ) + 1 O 2 ( g ) H 2 O( g ) r H = 241 .8 kJ mol 1
2

Solution: We need f H for the reaction

( 4) 2B ( s ) + 3H 2 ( g ) B 2 H 6 ( g )
reaction(4 ) = reaction(2 ) + 3 reaction(3 ) - reaction(1 )
Thus, f H = r H {reaction(2 )} + 3 r H {reaction(3 )} r H {reaction(1 )}
= { 2368 + 3 ( 241 .8 ) ( 1941 )}K J mol 1 = 1152 K J mol 1

28
2.32(b) When 2.25 mg of anthracene, C14H10(s), was burned in a bomb calorimeter the temperature rose by
1.35 K. Calculate the calorimeter constant. By how much will the temperature rise when 135 mg of phenol,
C6H5OH(s), is burned in the calorimeter under the same conditions?
Solution: For anthracene the reaction is
33
C14 H10 ( s) + O 2 ( g ) 14CO 2 ( g ) + 5H 2 O( l )
2
5
cU = c H ng RT [ 26 ] ng = - mol
2
1
c H = 7163 kJ mol ( Hand book of chemist ry and phy sics )

5
cU = 7163 kJ mol 1 - 8.3 10 3 kJ K -1 mol 1 298 K (assume T = 298 K)
2
1
= -7157 kJ mol
2.25 10 3 g
q = qV = ncU = (
7157 kJ mol 1
1
)
172.23 g mol
= 0.0935kJ
q 0.0935 kJ
C= = = 0.0693 kJ K 1 = 69.3 J K 1
T 1.35 K
15
When phenol is used the reaction is C6 H 5 OH ( s) + O 2 ( g ) 6CO 2 ( g ) + 3H 2 O( l )
2
c H = 3054 kJ mol 1 ( Table2.5 )
3
cU = c H ng RT , ng = - mol
2
( 3
) ( )
= 3054 kJ mol 1 + 8.314 10 3 kJ K -1 mol 1 ( 298 K )
2
= 3050 kJ mol 1
135 10 3 g
q =
94.12 g mol 1
( )
3050 kJ mol 1 = 4.37 5 kJ

q 4.37 5 kJ
T = = = +63 .1 K
C 0.0693 kJ K 1
Comment. In this case cU and c H differed by 0.1 per cent. Thus, to within 3 significan t
figures, it would not have mattered if we had used c H instead of cU , but for very precise work
it would.

29
2.35(b) Given that the standard enthalpy combustion of graphite is -393.51 kJ mol -1 and that of diamond is
-395.41kJ mol-1, calculate the enthalpy of the graphitediamond transition.
Solution: The difference of the equations is C( gr ) C( d )

trans H = [ 393.51 ( 395.41 ) ] kJ mol 1 = +1.90 kJ mol 1

30
2.39(b) Use standard enthalpies of formation to calculate the standard enthalpies of the following reactions:

( a) Cyclopropa ne ( g ) propene ( g )
( b ) HCl ( aq ) + NaOH ( aq ) NaCl ( aq ) + H 2O( l )
Solution:
( a ) r H = f H ( propene , g ) f H ( cycloropan e , g ) = [ ( 20 .42 ) ( 53 .30 ) ]kJ mol 1
= 32.88 kJ mol 1
( b ) The met ionic reaction is obtained from
H + ( aq ) + Cl ( aq ) + Na + ( aq ) + OH ( aq ) Na + ( aq ) + Cl ( aq ) + H 2 O( l )
and is H + ( aq ) + OH ( aq ) H 2 O( l )
( ) (
r H = f H ( H 2 O, l ) f H H + , aq f H OH , aq )
= [ ( 285 .83 ) ( 0 ) ( 229 .99 ) ] kJ mol 1

= 55 .84 kJ mol 1

31
2.40(b) Given the reactions(1) and (2) below, determine (a) r H and rU for reaction (3), (b)

f H for both HI(g) and H2O(g) all at 298 K. Assume all gases are perfect.

(1) H 2 ( g ) + I 2 ( g ) 2HI ( g ) r H = +52 .96 kJ mol 1


( 2) 2H 2 ( g ) + O 2 ( g ) 2H 2O( g ) r H = 483 .64 kJ mol 1
( 3) 4HI ( g ) + O 2 ( g ) I 2 ( s) + 2H 2O( g )
Solution:
reaction ( 3) = reaction ( 2) 2( reaction (1) )
( a ) r H ( 3) = r H ( 2) 2 r H (1)
(
= 483 .64 kJ mol 1 2 52 .96 kJ mol 1 )
1
= -589.56kJ mol
rU
= r H ng RT
(
= -589.56kJ mol 1 ( 3) 8.314 J K -1 mol 1 ( 298 K ) )
= -589.56kJ mol + 7.43 kJ mol = 582 .13 kJ mol 1
1 1

( )
( b ) f H ( HI ) = 1 52 .96 kJ mol 1 = 26 .48 kJ mol 1
2
1
( )
f H ( H 2O ) = 483 .64 kJ mol 1 = 241 .82 kJ mol 1
2

32
2.44(b) Calculate r H and r U at 298 K and r H at 348 K for the hydrogenation of ethyne

(acetylene)to ethane (ethylene) from the enthalpy of combustion and heat capacity data in Tables 2.5 and
2.6. Assume the heat capacities to be constant over the temperature range involved.
Solution: The hydrogenation reaction is

( 1) C 2 H 2 ( g ) + H 2 ( g ) C 2 H 4 ( g ) r H (Tc ) = ?

The reactions and accompanying data which are to be combined in order to yield reaction (1) and
Are

( 2 ) H 2 ( g ) + 1 O 2 ( g ) H 2 O( l ) C H ( 2) = 285.83 kJ mol 1
2
( 3)C 2 H 4 ( g ) + 3O 2 ( g ) 2H 2 O( l ) + 2CO 2 ( g ) C H ( 3) = 1411 kJ mol 1

( 4)C 2 H 2 ( g ) + 5 O 2 ( g ) H 2 O( l ) + 2CO 2 ( g )
C H ( 4 ) = 1300 kJ mol 1

2
reaction(1 ) = reaction(2 ) - reaction(3 ) + reaction(4 )
Hence,
( a) r H (T ) = r H ( 2) c H ( 3) + c H ( 4)
= {( 285 .83 ) ( 1411 ) + ( 1300 )}kJ mol 1
1
175 kJ mol
rU (T ) = r H (T ) n g RT [ 26 ] g = -1
(
= - 175kJ mol 1
+ 2.48 kJ mol 1
) = 173 kJ mol 1

( b ) r H ( 348 K ) = r H ( 298 K ) r C p ( 348 K 298 K )[ Example2.7 ]


r C p = v J C p ,m ( J )[47 ] = C p ,m ( C 2 H 4 , g ) C p ,m ( C 2 H 2 , g ) C p ,m ( H 2 , g )
J

= ( 43 .56 43 .93 28 .82 ) 10 3 kJ K -1 mol 1


= 29 .19 10 3 kJ K -1 mol 1

(
r H ( 348 K ) = 175 kJ mol 1
) (29 .19 10 3
kJ K -1 mol 1
) ( 50 K )
1
= 176 kJ mol

33
Problems
2.2 An average human produces about 10MJ of heat each day through metabolic activity. If a human body
were an isolated system of mass 65 kg with the heat capacity of water, what temperature rise would the
body experience? Human bodies are actually open systems, and the main mechanism of heat loss is through
the evaporation of water. What mass of water should be evaporated each day to maintain constant
temperature?
Solution: Good approximate answers can be obtained from the data for the heat capacity and molar heat of
vaporization of water at 25.[Table 2.6 and 2.3]
C p ,m ( H 2O ) = 75 .3J K 1 mol 1
vap H ( H 2 O) = 44 .0kJ mol 1

65 kg
n( H 2 O ) = = 3.6 10 3 mol
0.018 kg mol 1
From H = nC p,m T , we obtain
H 1.0 10 4 kJ
T = =
nC p,m ( ) (
3.6 10 3 mol 0.0753 kJ K -1 mol 1
) = +37 K
m
From H = nvap H = vap H
M

m=
M H
=
( ) (
0.018 kg mol 1 1.0 10 4 kJ
= 4.09 kg
)

vap H 44.0kJ mol 1

Comment: This estimate would correspond to about 30 glasses of water per day, which is much higher than
the average consumption. The discrepancy may be a result of our assumption that evaporation of water is
the main mechanism of heat loss.

34
Chapter 3 The First Law: the machinery
Exersises
3.10(b) A vapour at 22 atm and 5 was allowed to expand adiabatically to a final pressure of 1.00 atm; the
temperature fell by 10 K. Calculate the Joule-Thomson coefficient, , at 5, assuming it remains constant
over this temperature range.
Solution:
The Joule-Thomson coefficient is the ratio of temperature change to pressure change under conditions
of isenthalpic expansion. So
T T 10 K
=
= = 0.48 K atm 1
p H p ( 1.00 22 ) atm

35
3.11(b) For a van der Waals gas, T = a 2 . Calculate U m for the isothermal reversible expansion of
Vm
argon from an initial volume of 1.00 L to 22.1 L at 298 K. What are the values of q and w?
Solution:
dU m U m
U m = U m (T,V m )dU m = dT +
V
dV m
T Vm m
dT = 0 in an isothermal process, so
U m a
dU m =
V
dV m = V 2 dV m
m T m

Vm2 Vm2 a 22.1 L mol -1 dV a


U m = dU m = 2
dV m = a -1
m
2
= 2 22 .1L mol -1
1.00 L mol -1
Vm1 m Vm1 m
Vm 1.00 L mol
Vm Vm
a a 21.1 a
= -1
+ -1
= = 0.954 75 L mol -1
22 .1L mol 1.00 L mol 22 .1L mol -1
a = 1.345 atm L2 mol 2
( ) (
U m = 0.95475 mol L1 1.345 atm L2 mol 2 )
(
= 0.28 41 atm L2 mol 2 ) (1.01325 10 5 Pa atm 1 )
1m 3
(
1.01325 10 5 Pa atm 1 3
10 L
)


= 130P am mol = 130 J mol 1
3 -1

RT a
w = pex dVm and p = for a van der Waals gas
Vm b Vm 2
RT a
so w = dVm + V 2 dVm = q + U m
V b
m m

Thus
22.1 L mol -1 RT
q = + dV m = + RT ln (Vm b )
-1

-1

V b
22 .1 L mol
1 .00 L mol -1
1.00 L mol
m
22 .1 3.22 10 2
( )
= + 8.314 J K -1 mol -1 ( 298 K ) ln 2
1.00 3.22 10
= +7.74 69 kJ mol -1
w = 774 7 J mol -1 + 130 J mol -1 = 761 7 J mol -1 = 7.62 kJ mol -1

36
3.12(b) The volume of a certain liquid varies with temperature as

{
V = V ' 0.77 + 3.7 10 4 ( T/K ) + 1.52 10 6 ( T/K )
2
}
Where V ' is its volume at 298 K. Calculate its expansion coefficient, , at 310 K
Solution:
The expansion coefficient is

=
1 V
=
(
V' 3.7 10 4 K 1 + 2 1.52 10 6 T K 2 )
V T p V
[ ]
V' 3.7 10 4 K 1 + 2 1.52 10 6 ( T/K ) K 1
=
[
V ' 0.77 + 3.7 10 ( T/K ) + 1.52 10 ( T/K ) 2 ]
4 6

=
[3.7 10 4 K 1 + 2 1.52 10 6 ( 310 ) ] K 1 = 1.27 10 3 K 1
0.77 + 3.7 10 4 ( 310 ) + 1.52 10 6 ( 310 )
2

37

Вам также может понравиться